OG12 #87

This topic has expert replies
Newbie | Next Rank: 10 Posts
Posts: 7
Joined: Sun Feb 19, 2012 6:28 am

OG12 #87

by GMATbeater12 » Sat Mar 10, 2012 12:34 am
Could someone please help me with the following problem solving problem? It is problem 87 on page 164 of the Official Guide, 12th edition.

87. A necklace is made by stringing N individual beads together in the repeating pattern red bead, green bead, white bead, blue bead, and yellow bead. If the necklace design begins with a red bead and ends with a white bead. then N could equal

(A) 16
(B) 32
(C) 41
(D) 54
(E) 68


I answered A because I figured that you could have RGWBYRGW, but the answer is E. Why?

Legendary Member
Posts: 581
Joined: Sun Apr 03, 2011 7:53 am
Thanked: 52 times
Followed by:5 members

by killer1387 » Sat Mar 10, 2012 1:07 am
GMATbeater12 wrote:Could someone please help me with the following problem solving problem? It is problem 87 on page 164 of the Official Guide, 12th edition.

87. A necklace is made by stringing N individual beads together in the repeating pattern red bead, green bead, white bead, blue bead, and yellow bead. If the necklace design begins with a red bead and ends with a white bead. then N could equal

(A) 16
(B) 32
(C) 41
(D) 54
(E) 68


I answered A because I figured that you could have RGWBYRGW, but the answer is E. Why?
Check the pattern
i.e. (RGWBY)(RGWBY)......A times (RGW)
=> 5A+3
68 SATISFIES (for A=13)
HENCE E is the answer.

User avatar
Master | Next Rank: 500 Posts
Posts: 425
Joined: Wed Dec 08, 2010 9:00 am
Thanked: 56 times
Followed by:7 members
GMAT Score:690

by LalaB » Sat Mar 10, 2012 5:54 am
GMATbeater12 wrote:
I answered A because I figured that you could have RGWBYRGW,
using the same logic, u can figure out that B is also ok ;) 32/8=4

see the q.stem. it is said that u have a repeating patern -RGWBY, and need to end with W
so ur pattern is RGWBY + RGW . it means that u have 5x+3
now after checking each answer choice, u will find out that only option E is correct
5x+3=68 ;x=13
Happy are those who dream dreams and are ready to pay the price to make them come true.(c)

In order to succeed, your desire for success should be greater than your fear of failure.(c)

Newbie | Next Rank: 10 Posts
Posts: 7
Joined: Sun Feb 19, 2012 6:28 am

by GMATbeater12 » Sat Mar 10, 2012 3:48 pm
Thank you for your help.

I still do not understand it, though. I see how if 16 were be correct, so would 32, but both fulfill the requirement that the last bead is white. Why is that not correct?

Legendary Member
Posts: 581
Joined: Sun Apr 03, 2011 7:53 am
Thanked: 52 times
Followed by:5 members

by killer1387 » Sat Mar 10, 2012 6:12 pm
GMATbeater12 wrote:Thank you for your help.

I still do not understand it, though. I see how if 16 were be correct, so would 32, but both fulfill the requirement that the last bead is white. Why is that not correct?
87. A necklace is made by stringing N individual beads together in the repeating pattern red bead, green bead, white bead, blue bead, and yellow bead. If the necklace design begins with a red bead and ends with a white bead. then N could equal

(A) 16
(B) 32
(C) 41
(D) 54
(E) 68

The part in bold is not correct. In any case 16 is not correct.

you need to follow the pattern (RGWBY) start with red till white.

see:possible arrangements are

(RGWBY)(RGW)--> 8 beads
(RGWBY)(RGWBY)(RGW)--> 13 beads
(RGWBY)(RGWBY)(RGWBY)(RGW)--> 18 beads
etc.

it is basically

(RGWBY)--> 5 beads
suppose you repeat it A times i.e. 5A beads

(RGW)--> 3 beads
hence number of beads will always satisfy 5A+3

put A=1,2,3,4.......infinity
i.e. 8, 13, 18, 23, 28.......

for A=13 its 68 beads

hence E.

HTH

Newbie | Next Rank: 10 Posts
Posts: 7
Joined: Sun Feb 19, 2012 6:28 am

by GMATbeater12 » Sat Mar 10, 2012 6:55 pm
Ok, I see it now. Thank you!